Why Does the Series Sum of (ln n)/n Exceed Sum of 1/n?

  • Thread starter Thread starter transgalactic
  • Start date Start date
  • Tags Tags
    Expression
Click For Summary
The series sum of (ln n)/n exceeds the sum of 1/n because ln n grows larger than 1 for almost all positive n, despite ln 1 being 0. When comparing these series, it is essential to recognize that the sum can be split into parts, allowing for clearer analysis of convergence. The discussion emphasizes that in infinite series, ignoring a finite number of terms does not impact convergence or divergence. Therefore, the behavior of ln n as n increases is crucial in understanding the divergence of the series. The conclusion is that the growth rate of ln n ensures that the series sum of (ln n)/n diverges more significantly than that of 1/n.
transgalactic
Messages
1,386
Reaction score
0
<br /> \sum_{n=1}^{\infty}\frac{\ln n}{n}&gt; \sum_{n=1}^{\infty}\frac{1}{n}<br />

ln is not always bigger then 1
so when i am doing the comparing test
i can't use that
because ln 1 =0

??
 
Last edited:
Physics news on Phys.org
ln n > 1 for almost every positive number n. Compare the graphs of y = ln x and y = 1 and you'll see that what I'm saying is true.
 
you said yourself "almost" not absolutely
 
Remember that that sum is really just a whole list of terms, all added up. So you can split the sum into two sums, or write out some of the terms explicitly if you want. Just to give an example:

\sum_{n=0}^{\infty} n^2 e^{-n} = \left(\sum_{n=0}^{6} n^2 e^{-n}\right) + \left(\sum_{n=7}^{\infty} n^2 e^{-n}\right) = 0^2 e^{-0} + 1^2 e^{-1} + 2^2 e^{-2} + \sum_{n=3}^{\infty} n^2 e^{-n}

You can do something like this for one or both of the sums in your expression, and it should help.
 
transgalactic said:
you said yourself "almost" not absolutely
In an infinite series, you can always ignore a finite number of terms without affecting whether the series converges or diverges.
 
Question: A clock's minute hand has length 4 and its hour hand has length 3. What is the distance between the tips at the moment when it is increasing most rapidly?(Putnam Exam Question) Answer: Making assumption that both the hands moves at constant angular velocities, the answer is ## \sqrt{7} .## But don't you think this assumption is somewhat doubtful and wrong?

Similar threads

  • · Replies 3 ·
Replies
3
Views
1K
  • · Replies 3 ·
Replies
3
Views
2K
  • · Replies 7 ·
Replies
7
Views
2K
  • · Replies 6 ·
Replies
6
Views
2K
  • · Replies 14 ·
Replies
14
Views
3K
Replies
12
Views
3K
  • · Replies 5 ·
Replies
5
Views
2K
  • · Replies 6 ·
Replies
6
Views
2K
Replies
15
Views
2K
  • · Replies 1 ·
Replies
1
Views
2K